Đến nội dung

KietLW9 nội dung

Có 1000 mục bởi KietLW9 (Tìm giới hạn từ 12-05-2020)



Sắp theo                Sắp xếp  

#726644 xy+yz+xz=1. Tìm Min $\sum \frac{1}{4x^{2...

Đã gửi bởi KietLW9 on 08-05-2021 - 10:08 trong Bất đẳng thức và cực trị

Cho x, y, z > 0; xy+yz+xz=1. Tìm giá trị nhỏ nhất: $\frac{1}{4x^{2}-yz+2}+\frac{1}{4y^{2}-xz+2}+\frac{1}{4z^{2}-xy+2}$

Ta có: 

$\frac{1}{4x^{2}-yz+2}+\frac{1}{4y^{2}-xz+2}+\frac{1}{4z^{2}-xy+2}-1=\sum_{cyc}\frac{(x-y)^2(z^2+2xy+2)}{(4x^2-yz+2)(4y^2-zx+2)}\geqslant 0$




#735473 Với mỗi số nguyên dương $k$, ta định nghĩa $S(k)$ là tổng...

Đã gửi bởi KietLW9 on 26-10-2022 - 13:31 trong Số học

 Với mỗi số nguyên dương $k$, ta định nghĩa $S(k)$ là tổng các ước nguyên tố phân biệt của $k$. Ví dụ $S(20)=7,S(2)=2,S(1)=0$. Tìm tất cả các số nguyên dương $n$ để $S(2^n+1)=S(n)$




#725579 Với a, b là các số thực dương. Tìm MIN: $A=\sqrt{\frac...

Đã gửi bởi KietLW9 on 18-04-2021 - 07:39 trong Bất đẳng thức và cực trị

Ta có: $\sqrt{\frac{a^3}{a^3+8b^3}}-\frac{a^2}{a^2+2b^2}=\frac{\frac{4a^3b^2(a-b)^2}{(a^3+8b^3)(a^2+2b^2)^2}}{\sqrt{\frac{a^3}{a^3+8b^3}}+\frac{a^2}{a^2+2b^2}}\geqslant 0\Rightarrow \sqrt{\frac{a^3}{a^3+8b^3}}\geqslant \frac{a^2}{a^2+2b^2}$ (1)

          $\sqrt{\frac{4b^3}{b^3+(a+b)^3}}-\frac{2b^2}{a^2+2b^2}=\frac{\frac{4b^3(a-b)^2(a^2+ab+2b^2)}{[b^3+(a+b)^3](a^2+2b^2)^2}}{\sqrt{\frac{4b^3}{b^3+(a+b)^3}}+\frac{2b^2}{a^2+2b^2}}\geqslant 0$ (2)

Cộng theo vế hai bất đẳng thức (1) và (2), ta được: $\sqrt{\frac{a^3}{a^3+8b^3}}+\sqrt{\frac{4b^3}{b^3+(a+b)^3}}\geqslant \frac{a^2+2b^2}{a^2+2b^2}=1$

Đẳng thức xảy ra khi $a = b>0$




#726405 Với $x>1$ .Tìm $MinM=\frac{1+x^{4}...

Đã gửi bởi KietLW9 on 03-05-2021 - 21:37 trong Bất đẳng thức và cực trị

Với $x>1$ .Tìm $MinM=\frac{1+x^{4}}{x(x-1)(x+1)}$

$\frac{(x^4+1)^2}{x^2(x-1)^2(x+1)^2}-8=\frac{(x^4-4x^2+1)^2}{x^2(x-1)^2(x+1)^2}\geqslant 0$




#724684 Với $a\neq b\neq c$ chứng minh $\sum \left...

Đã gửi bởi KietLW9 on 28-03-2021 - 09:54 trong Đại số

Ta dễ có đẳng thức: $\frac{a+b}{a-b}.\frac{b+c}{b-c}+\frac{b+c}{b-c}.\frac{c+a}{c-a}+\frac{c+a}{c-a}.\frac{a+b}{a-b}=-1$

Ta luôn có: $(\frac{a+b}{a-b}+\frac{b+c}{b-c}+\frac{c+a}{c-a})^2\geq 0$

$\Leftrightarrow \sum (\frac{a+b}{a-b})^2+2(\frac{a+b}{a-b}.\frac{b+c}{b-c}+\frac{b+c}{b-c}.\frac{c+a}{c-a}+\frac{c+a}{c-a}.\frac{a+b}{a-b})\geq 0$

Vậy $\sum (\frac{a+b}{a-b})^2\geq 2(Q.E.D)$

Đẳng thức xảy ra khi $\frac{a+b}{a-b}+\frac{b+c}{b-c}+\frac{c+a}{c-a} = 0$




#726671 Với $-4< x< 9$ , tìm min của $P = \frac{1...

Đã gửi bởi KietLW9 on 08-05-2021 - 17:09 trong Bất đẳng thức và cực trị

Với $-4< x< 9$ , tìm min của $P = \frac{1}{9-x} + \frac{1}{x+4}$

Ta có: 

$\frac{1}{9-x} + \frac{1}{x+4}-\frac{4}{13}=\frac{(2x-5)^2}{13(x+4)(9-x)}\geqslant 0$




#732013 Về việc thiết lập các câu hỏi tương tự

Đã gửi bởi KietLW9 on 14-12-2021 - 11:52 trong Góp ý cho diễn đàn

Em xin có ý kiến là diễn đàn nên có chức năng câu hỏi tương tự được không ạ. Ví dụ như một bạn đăng câu hỏi thì nó sẽ hiện ra một list các câu hỏi tương tự từ mấy năm trước để các bạn có thể nhận được câu trả lời nhanh nhất và cũng đỡ trùng lặp nhiều câu hỏi ạ. Chức năng này em nhớ là có bên OLM và hoc24h cũng rất hiệu quả. Nếu được thì em cảm ơn ạ! 




#726575 Về một bài toán trong Hello IMO 2007

Đã gửi bởi KietLW9 on 07-05-2021 - 14:36 trong Tài liệu, chuyên đề, phương pháp về Bất đẳng thức

Đặt $p=x+y+z,q=xy+yz+zx,r=xyz$

Áp dụng bất đẳng thức AM-GM ta được: $2xyz+1=xyz+xyz+1\geq 3\sqrt[3]{x^{2}y^{2}z^{2}}\geq \frac{9xyz}{x+y+z}$ ( Do $(x+y+z)^3\geqslant 27xyz$)

Vậy ta có $ 2r+1\geq \frac{9r}{p}$

Ta có: $p^{3}\geq 27r$

Theo Schur ta có: $r\geq \frac{p(4q-p^{2})}{9}\Rightarrow 4q\leq p^{2}+\frac{9r}{p}$

Bất đẳng thức cần chứng minh tương đương với :

$2(p^{2}-2q)+r+8-5p\geq 0\Leftrightarrow 4(p^{2}-2q)+(2r+1)+15-10p\geq 0$

$\Leftrightarrow 4(p^{2}-2q)+\frac{9r}{p}+15-10p\geq 0\Leftrightarrow 4p^{2}-2p^{2}-\frac{9r}{p}+15-10p\geq 0$

$\Leftrightarrow 2p^{3}-9r-10p^{2}+15p\geq 0\Leftrightarrow 2p^{3}-\frac{p^{3}}{3}-10p^{2}+15p\geq 0\Leftrightarrow 5p^{3}-30p^{2}+45p\geq 0$

$\Leftrightarrow p(p-3)^{2}\geq 0$

Bất đẳng thức cuối luôn đúng nên ta có điều phải chứng minh




#724822 Về một bài toán trong Hello IMO 2007

Đã gửi bởi KietLW9 on 01-04-2021 - 18:57 trong Tài liệu, chuyên đề, phương pháp về Bất đẳng thức

Xét biểu thức: $6(2(x^2+y^2+z^2)+xyz+8-5(x+y+z))=12(x^2+y^2+z^2)+6xyz+48-30(x+y+z)\geqslant12(x^2+y^2+z^2)+3(xyz+xyz+1)+45-5((x+y+z)^2+9)\geqslant 7(x^2+y^2+z^2)+9\sqrt[3]{x^2y^2z^2} -10(xy+yz+zx)=7(x^2+y^2+z^2)+\frac{9xyz}{\sqrt[3]{xyz}} -10(xy+yz+zx)\geqslant 7(x^2+y^2+z^2) +\frac{27xyz}{x+y+z}-10(xy+yz+zx)$

Áp dụng Schur, ta có: $(x+y+z)^3+9xyz\geqslant 4(x+y+z)(xy+yz+zx) \Leftrightarrow \frac{9xyz}{x+y+z}\geqslant 2(xy+yz+zx)-(x^2+y^2+z^2) $

Do đó: $7(x^2+y^2+z^2) +\frac{27xyz}{x+y+z}-10(xy+yz+zx)\geqslant 7(x^2+y^2+z^2)+6(xy+yz+zx)-3(x^2+y^2+z^2)-10(xy+yz+zx)=4(x^2+y^2+z^2-xy-yz-zx)\geqslant 0 $

Vậy chứng minh hoàn tất

Đẳng thức xảy ra khi x = y = z = 1




#731942 VMO ĐẮC NÔNG 2021-2022

Đã gửi bởi KietLW9 on 09-12-2021 - 20:08 trong Tài liệu - Đề thi

Câu cực trị sao mình thấy dễ thế nhỉ?

$P\geqslant \sum \frac{ab(a+b)}{2ab}=\sum \frac{a+b}{2}=a+b+c=2022$




#724903 Vietnam TST 2021

Đã gửi bởi KietLW9 on 04-04-2021 - 10:04 trong Thi HSG Quốc gia và Quốc tế

Bài 4 (7 điểm): Cho các số thực không âm $a,b,c$ thoả mãn $2 \left ( a^2 +b^2 + c^2 \right ) +3(ab+bc+ca)=5(a+b+c)$.

Chứng minh rằng $4\left ( a^2 +b^2 + c^2 \right ) +2(ab+bc+ca)+7abc \leq 25$ .

Đặt $a+b+c=p,ab+bc+ca=q,abc=r$ thì giả thiết trở thành $2(p^2-2q)+3q=5p\Rightarrow q=p(2p-5)$ và ta cần chứng minh: $4p^2-6q+7r\leqslant 25$ 

Do a, b, c không âm nên $q\geqslant 0\Rightarrow p(2p-5) \geqslant 0$  suy ra hoặc $p = 0$ hoặc $p\geqslant \frac{5}{2}$ (1)

Theo một đánh giá quen thuộc: $3q\leqslant p^2 \Rightarrow p(2p-5)=q\leqslant \frac{p^2}{3} \Rightarrow 5p(p-3)\leqslant 0\Rightarrow 0\leqslant p\leqslant 3 $  (2)

Từ (1) và (2) suy ra $p = r = 0$ hoặc $\frac{5}{2}\leqslant p\leqslant 3$  

* Xét $p = 0$ thì $q = 0$ và bất đẳng thức hiển nhiên đúng

* Xét $\frac{5}{2}\leqslant p\leqslant 3$ thì ta có:  $4p^2-6q+7r-25\leq 4p^2-6p(2p-5)+7.\frac{pq}{9}-25=\frac{14}{9}p^3-\frac{107}{9}p^2+30p-25=\frac{1}{9}(p-3)(p-\frac{5}{2})(14p-30)\leqslant 0$*luôn đúng với mọi p thuộc $[\frac{5}{2},3]$*

$\Rightarrow 4p^2-6q+7r\leqslant 25 (Q.E.D)$

Vậy bất đẳng thức được chứng minh

Đẳng thức xảy ra khi a = b = c = 1 :D




#725700 Trên cạnh $AC,BC$ của $\Delta ABC$ theo thứ tự lấy c...

Đã gửi bởi KietLW9 on 21-04-2021 - 08:16 trong Hình học

$\boxed{Problem 13}$Trên cạnh $AC,BC$ của $\Delta ABC$ theo thứ tự lấy các điểm $M$ và $K$, trên đoạn thẳng $MK$ lấy điểm $P$ sao cho $\frac{AM}{MC}=\frac{CK}{KB}=\frac{MP}{PK}$. Tính diện tích $\Delta ABC$, nếu diện tích tam giác $AMP$ và $BKP$ bằng $S_1,S_2$.




#725727 Trên các cạnh $AB,CD$ của hình vuông $ABCD$ lần lượt lấy...

Đã gửi bởi KietLW9 on 21-04-2021 - 13:58 trong Hình học

$\boxed{Problem 17}$Trên các cạnh $AB,CD$ của hình vuông $ABCD$ lần lượt lấy các điểm $M,N$ sao cho $AM=CN=\frac{AB}{3}$. Gọi $K$ là giao điểm của $AN$ và $DM$. Chứng minh rằng trực tâm của tam giác $ADK$ nằm trên cạnh $BC$.




#725695 Trên các cạnh $AB,BC,CA$ của $\Delta ABC$ lần lượt l...

Đã gửi bởi KietLW9 on 21-04-2021 - 07:53 trong Hình học

$\boxed{Problem 10}$Trên các cạnh $AB,BC,CA$ của $\Delta ABC$ lần lượt lấy các điểm $M,N,P$ sao cho $\Delta MNP$ đều. Biết các tam giác $AMP,BMN,CNP$ có diện tích bằng nhau. Chứng minh rằng: $\Delta ABC$ đều.




#725806 Trong các tứ giác với hai đường chéo có độ dài đã cho và góc giữa hai đường c...

Đã gửi bởi KietLW9 on 23-04-2021 - 19:08 trong Hình học

$\boxed{Problem 25}$ Trong các tứ giác với hai đường chéo có độ dài đã cho và góc giữa hai đường chéo có độ lớn đã cho, xác định tứ giác có chu vi nhỏ nhất.




#725606 TOPIC VỀ CÁC BÀI HÌNH HỌC LỚP 7,8

Đã gửi bởi KietLW9 on 18-04-2021 - 20:28 trong Hình học

$\boxed{1}$Cho hình vuông $ABCD$, $I$ là một điểm bất kì trên cạnh $AB$ ($I$ khác $A$ và $B$). Tia $DI$ cắt $CB$ tại $E$. Đường thẳng $CI$ cắt $AE$ tại $M$. Chứng minh rằng $DE$ vuông góc với $BM$ 

$\boxed{2}$Cho hình chữ nhật $ABCD$ có $AB=a,AD=b$ $(a>b>0)$. Tia phân giác của $\widehat{BAD}$ cắt $BD,CD$ lần lượt tại $E,K$. Trên cạnh $BD$ lấy điểm $H$ sao cho $AE$ là phân giác của $\widehat{CAH}$. Gọi $F$ là giao điểm của $HK$ và $AB$. Chứng minh rằng: $C,E,F$ thẳng hàng.

$\boxed{3}$Cho tam giác nhọn $ABC$ ($AB<AC$), các đường cao $BD$ và $CE$. $DE$ cắt $BC$ tại $K$. Các tia phân giác của các góc $BAC,DKB$ cắt nhau tại $S$. $M,N$ lần lượt là trung điểm của $BD,CE$. Chứng minh rằng $M,S,N$ thẳng hàng.

$\boxed{4}$Trên cạnh $AC,BC$ của tam giác ABC theo thứ tự lấy $M,K$, trên đoạn thẳng $MK$ lấy điểm $P$ sao cho $\frac{AM}{MC}=\frac{CK}{KB}=\frac{MP}{PK}$. Tính diện tích tam giác ABC, nếu diện tích tam giác AMP và BKP bằng S1, S2.




#734439 TOPIC [MỘT SỐ BÀI TOÁN SỐ HỌC]

Đã gửi bởi KietLW9 on 16-08-2022 - 15:14 trong Số học

$\textbf{Bài toán 3 (THTT T10/517).}$ Tìm các số nguyên dương $k$ và số nguyên tố $p$ thỏa mãn $$k!=(p^3-1)(p^3-p)(p^3-p^2)$$

$\textbf{Lời giải.}$ Xét $p=2$ thì $k!=168$. Trường hợp này không có số $k$ nào thỏa mãn.

Xét $p>2$ thì $k!=p^3(p-1)^3(p+1)(p^2+p+1)$ nên $v_p(k!)=3$

Mặt khác theo công thức Lengendre thì $v_p(k!)=\sum_{i=1}^{n}\left [ \frac{k}{p^i} \right ]$ với $n$ là số thỏa mãn $p^n\leq k< p^{n+1}$

Nếu $n\geq 2$ thì $k\geq p^2\geq 3p$. Nếu $n=1$ thì $3=\left [ \frac{k}{p} \right ]\Rightarrow k\geq 3p$. Vì thế ta luôn có $k\geq 3p>2(p+1)\Rightarrow (p+1)^2|k!$

$\Rightarrow p+1|p^3(p-1)^3(p^2+p+1)$

Mà $(p+1,p^2+p+1)=1$ nên $p^3(p-1)^3\equiv 0(\text{mod p+1})$

Mà $p^3(p-1)^3\equiv (-1)^3.(-2)^3\equiv 8(\text{mod p+1})\Rightarrow p+1|8$ nên $p=7$ hoặc $p=3$

Hai số này đều không thỏa nên ta kết luận không tồn tại số nguyên dương $k$ và số nguyên tố $p$ thỏa mãn đề bài




#734970 TOPIC [MỘT SỐ BÀI TOÁN SỐ HỌC]

Đã gửi bởi KietLW9 on 15-09-2022 - 23:24 trong Số học

$\textbf{Bài toán 5.}$ Cho $n$ là số nguyên dương lớn hơn $1$ sao cho $n|6^n+7^n$. Chứng minh rằng $13|n$

$\textbf{Lời giải.}$ Rõ ràng $(n,42)=1$. Gọi $p$ là ước nguyên tố nhỏ nhất của $n$

Vì $(p,7)=1$ nên tồn tại số nguyên $p'$ sao cho $7p'\equiv 1(\text{mod p})$

Ta có: $6^n\equiv -7^n(\text{mod p})\Rightarrow (6p')^n\equiv -(7p')^n\equiv -1(\text{mod p})\Rightarrow (6p')^{2n}\equiv 1(\text{mod p})$

Gọi $h=\text{ord}_p(6p')$ thì $h|p-1$ và $h|2n$

Nếu $h$ lẻ thì $h<p$ và $h|n$ nên $h=1$

Nếu $h$ chẵn thì $h=2l$ suy ra $l<p$ và $l|n$ do đó $l=1$ và $h=2$

Vậy ta đều có $(6p')^2\equiv 1(\text{mod p})\Rightarrow (6^2-7^2)p'^2\equiv 0(\text{mod p})$

Mà rõ ràng $(p',p)=1$ do đó $p=13$

Vậy $13|n$




#734437 TOPIC [MỘT SỐ BÀI TOÁN SỐ HỌC]

Đã gửi bởi KietLW9 on 16-08-2022 - 14:07 trong Số học

$\textbf{Bài toán 2 (THTT T10/516).}$ Tìm tất cả các cặp số nguyên tố $p,q$ để $\frac{3^q+1}{p^2+1}$ và $\frac{3^p+1}{q^2+1}$ đồng thời là các số nguyên dương

 

$\textbf{Lời giải.}$ Không mất tính tổng quát, ta có thể giả sử $p\leq q$

 

$\textbf{Trường hợp 1.}$ $p=2$ khi đó $5|3^q+1$ và $q^2+1|10$ nên dễ dàng tìm được $q=2$

 

$\textbf{Trường hợp 2.}$ $p$ là số nguyên tố lẻ thì rõ ràng $p^2+1$ sẽ chứa thừa số nguyên tố lẻ. Gọi $r$ là một ước nguyên tố lẻ của $p^2+1$ thì $3^q+1\equiv 0(\text{mod r})\Rightarrow (-3)^{q}\equiv 1(\text{mod r})$

Gọi $h=ord_r(-3)$ thì $h|q$. Nếu $h=1$ thì ta suy ra vô lí ngay nên $h=q$ (Lưu ý rằng $p^2+1$ không chứa thừa số nguyên tố $3$ nên $(r,3)=1$)

Mà theo định lý Fermat nhỏ ta cũng có được $(-3)^{r-1}\equiv 1(\text{mod r})$ nên $q|r-1$. 

Như vậy mọi ước nguyên tố lẻ của $p^2+1$ đều đồng dư với $1$ theo modulo $q$

Mặt khác $p^2+1=2m$ ($m$ là số nguyên dương lẻ) nên $p^2+1\equiv 2(\text{mod q})\Rightarrow q|(p+1)(p-1)$

Do $p-1<p\leq q$ nên $q|p+1$. Lúc này ta có $p\leq q\leq p+1$ nên $q=p+1$ suy ra q là số nguyên tố chẵn (Vô lí)

Vậy $(p,q)=(2,2)$

$\textbf{Bài toán tương tự.}$ Tìm tất cả các cặp số nguyên tố $p,q$ để $\frac{(2p^2-1)^q+1}{p+q}$ và $\frac{(2q^2-1)^p+1}{p+q}$ đồng thời là các số nguyên dương




#734369 TOPIC [MỘT SỐ BÀI TOÁN SỐ HỌC]

Đã gửi bởi KietLW9 on 14-08-2022 - 07:05 trong Số học

$\textbf{Bài toán 1 (Brazil National Olympiad 2009).}$ Cho $p,q$ là các số nguyên tố thỏa mãn $q=2p+1$. Chứng minh rằng tồn tại một bội của $q$ mà tổng các chữ số trong biểu diễn thập phân của nó không vượt quá $3$.

$\textbf{Lời giải.}$

Rõ ràng $q\geq 5$. Xét $q=5$ thì $10$ là bội của $5$ và tổng của các chữ số của $10$ là $1$ (thỏa mãn)

Nếu $q>5$ thì $(q,10)=1$. Áp dụng định lý Fermat nhỏ, ta được: $10^{q-1}\equiv 1 (\text{mod q})\Rightarrow 10^{2p}\equiv 1 (\text{mod q})\Rightarrow q | (10^p+1)(10^p-1)$

$\textbf{Trường hợp 1.}$ $q|10^p+1$ thì rõ ràng $10^p+1$ là một bội của $q$ mà tổng các chữ số trong biểu diễn thập phân của nó là $2$ nên bài toán được chứng minh

$\textbf{Trường hợp 2.}$ $q|10^p-1$. Gọi $h=ord_q(10)$ thì $h=1$ hoặc $h=p$. Rõ ràng $h=1$ vô lí vì nếu $h=1$ thì $q|9$. Do đó $h=p$ hay nói cách khác $p$ là cấp của $10$ theo modulo $q$

Suy ra khi ta đem chia các số $10^1,10^2,...,10^p$ cho $q$ thì chúng sẽ lần lược nhận các số dư phân biệt theo thứ tự là $r_1,r_2,...,r_p$ với $1\leq r_i\leq 2p(i=\overline{1,p})$

Nếu $r_i=p$ thì $2.10^i+1\equiv 2r_i+1=2p+1\equiv 0(\text{mod q})$ nên đây cũng là một bội của $q$ có tổng các chữ số là $3$. Tương tự với $r_i=2p$ thì ta cũng thấy thỏa mãn

Trong trường hợp trong $p$ số trên không có số nào khi chia $q$ có số dư là $p$ hoặc $2p$ thì ta phân hoạch $2p-2$ số dư còn lại trong tập hợp số dư thành $p-1$ tập con như sau $$\left \{ 1,2p-1 \right \},\left \{ 2,2p-2 \right \},...,\left \{ p-1,p+1 \right \}$$

Có $p$ số mà chỉ có $p-1$ tập con nên theo nguyên lí Dirichlet thì tồn tại hai số $r_i,r_j$ sao cho $r_i+r_j=2p$. Tức là $10^i+10^j+1 \equiv 2p+1\equiv 0(\text{mod q})$. Mà $10^i+10^j+1$ là một số biểu diễn trong hệ thập phân có tổng các chữ số là $3$ nên trường hợp nãy vẫn thỏa mãn

Vậy ta có điều phải chứng minh




#734484 TOPIC [MỘT SỐ BÀI TOÁN SỐ HỌC]

Đã gửi bởi KietLW9 on 18-08-2022 - 10:46 trong Số học

$\textbf{Bài toán 4.}$ Tìm tất cả các cặp số nguyên dương $(a,b)$ sao cho $a,b>1$ và $b^a|a^b-1$

 

$\textbf{Lời giải.}$

$\textbf{Bổ đề.}$ Cho $a,b$ là các số nguyên, $n$ là số nguyên dương và $p$ là số nguyên tố thỏa mãn $(n,p-1)=1$. Khi đó nếu $p|a^n-b^n$ thì $ p|a-b$

Gọi $p$ là ước nguyên tố nhỉ nhất của $b$

Áp dụng bổ đề cho bài toán trên, ta được: $p|a-1$ do $p$ là ước nguyên tố nhỏ nhất của $b$ nên $(b,p-1)=1$

Nếu $p$ lẻ thì áp dụng bổ đề LTE, ta được: $v_p(a^b-1)=v_p(a-1)+v_p(b)\geqslant v_p(b^a)=av_p(b)\Rightarrow v_p(b)(a-1)\leq v_p(a-1)$

Mà $v_p(b)(a-1)\geq a-1$ nên $a-1\leq v_p(a-1)$. Đây là một điều hết sức vô lí nên $p=2$

Lúc đó $b$ chẵn và $a$ lẻ

Tiếp tục sử dụng bổ đề LTE cho $p=2$, ta được: $av_2(b)\leq v_2(a^b-1)=v_2(a+1)+v_2(a-1)+v_2(b)-1\Rightarrow a\leq v_2(b)(a-1)+1=v_2(a+1)+v_2(a-1)$

Điều này chỉ xảy ra khi $a=3$. Lúc đó $v_2(b)=1$. Đặt $b=2k$ với $k$ là số nguyên dương lẻ thì ta cần tìm số $k$ sao cho $(2k)^3|3^{2k}-1$

Nếu $k>1$ thì rõ ràng $k$ tồn tại ước nguyên tố lẻ nên gọi $q$ là ước nguyên tố lẻ nhỏ nhất của $k$. Khi đó theo bổ đề chứng minh ở trên thì $q|3^{2k}-1\Rightarrow q|9-1=8$. Vô lí do $q$ lẻ

Do đó $k=1$ suy ra $b=2$

Vậy $a=3,b=2$




#734373 TOPIC [MỘT SỐ BÀI TOÁN SỐ HỌC]

Đã gửi bởi KietLW9 on 14-08-2022 - 10:34 trong Số học

Được anh ạ




#735772 TOPIC [MỘT SỐ BÀI TOÁN SỐ HỌC]

Đã gửi bởi KietLW9 on 20-11-2022 - 01:33 trong Số học

$\textbf{Bài toán 7.}$ Chứng minh rằng với mọi $n$ nguyên dương luôn tồn tại $m$ để $$7^n|3^m+5^m-1$$

$\textbf{Lời giải.}$ Chọn $m=7^{n-1}$ và sử dụng LTE, ta có: $$v_7(8^m-1)=v_7(8-1)+v_7(m)=n\Rightarrow 7^n|8^m-1=(2^m-1)(4^m+2^m+1)$$

Mặt khác $m$ không là bội của $3$ nên $7^n \not| 2^m-1\Rightarrow 7^n|4^m+2^m+1$

Kết hợp với $m$ lẻ ta có: $v_7(4^m+3^m)=v_7(3+4)+v_7(m)=n\Rightarrow 4^m\equiv -3^m$ (mod $7^n$)

Tương tự ta cũng có: $2^m\equiv -5^m$ (mod $7^n$) nên $3^m+5^m-1\equiv 0$ (mod $7^n$)

$\textbf{Bài toán 8.}$ Cho $p$ là một số nguyên tố lẻ. Chứng minh rằng mọi ước nguyên tố của $\frac{p^{2p}+1}{p^2+1}$ đều có dạng $4k+1$

$\textbf{Lời giải.}$ Rõ ràng với $p$ lẻ thì $\frac{p^{2p}+1}{p^2+1}$ lẻ nên tất cả các ước nguyên tố của $\frac{p^{2p}+1}{p^2+1}$ đều lẻ. Gọi $q$ là một trong số chúng $\Rightarrow q|p^{4p}-1$

Gọi $h$ là cấp của $p$ theo modulo $q$ thì $h|4p$ nên $h \in \left \{ 1;2;4;p;2p;4p \right \}$

Nếu $h \in \left \{ 1;2;p;2p \right \}$ thì ta luôn có $p^{2p}\equiv 1(\text{mod q})$, vô lí với $p^{2p}\equiv -1(\text{mod q})$

* Nếu $h=4p$ thì $q\equiv 1(\text{mod 4p})$

* Nếu $h=4$ thì $q$ vẫn có dạng $4k+1$

Do đó ta có điều phải chứng minh

$\textbf{Bài toán 9.}$ Cho $a,m,n$ là các số nguyên lớn hơn $1$. Chứng minh rằng nếu $a^2+a+1|(a+1)^m+a^n$ thì $3|m+n$




#735043 TOPIC [MỘT SỐ BÀI TOÁN SỐ HỌC]

Đã gửi bởi KietLW9 on 21-09-2022 - 14:23 trong Số học

$\textbf{Bài toán 6.}$ Xét $p$ là số nguyên tố có dạng $4k+1$ trong đó $k$ là số nguyên dương thỏa mãn $p^2|2^{p-1}-1$. Gọi $q$ là ước nguyên tố lớn nhất của $2^p-1$ Chứng minh rằng: $2^q > (6p)^p$

 

$\textbf{Lời giải.}$ 

Ta có phân tích tiêu chuẩn của $2^p-1$ như sau: $2^p-1=q_{1}^{a_1}q_{2}^{a_2}...q_{k}^{a_k}$ trong đó $q_1,q_2,...,q_k$ là các ước nguyên tố lẻ phân biệt của $2^p-1$.

Đầu tiên ta sẽ chứng minh $q_i\equiv 1(\text{mod p}),\forall i=\overline{1,k}$

Thật vậy, ta có: $q_i|2^{q_i-1}-1$ nên $\left\{\begin{matrix}ord_{p_i}(2)|p & \\ ord_{p_i}(2)|q_i-1 & \end{matrix}\right.$ 

Từ đây dễ dàng suy ra $ord_{p_i}(2)=p$ nên ta có điều phải chứng minh

Đặt $q_i=m_ip+1$ trong đó $m_i \in \mathbb{Z}^+,\forall i =\overline{1,k}$

Kết hợp với $p^2|2^p-2$ ta suy ra $\prod_{i=1}^{k}(m_ip+1)^{a_i}\equiv 1$ (mod $p^2$). Mà $(m_ip+1)^{a_i}\equiv a_im_ip+1$ (mod $p^2$) nên $\prod_{i=1}^{k}(1+a_1m_ip)\equiv 1$ (mod $p^2$) nên $ \sum_{i=1}^{k}a_im_i\equiv 0$ (mod $p$)

Mà lại có $q_i|2^p-1\Rightarrow (\frac{2}{q_i})=1$ nên $q_i\equiv 1,7(\text{mod 8})$ do đó $8|m_i$ hoặc $6|m_ip$ nên $m_i\geq 6$

Xét $q$ là ước nguyên tố lớn nhất của $2^p-1$ sao cho $q=mp+1$ thì rõ ràng $m$ là chỉ số lớn nhất trong các số $m_i$ khi đó $2^q>2^{pm}>(2^p-1)^m>(6p)^{(a_1+a_2+...+a_k)m}>(6p)^{\sum_{i=1}^{k}a_im_i}\geq (6p)^p$

Vậy ta có điều phải chứng minh




#724703 Tim min A=$\sum \frac{x^3-y^3}{(x-y)^3}$

Đã gửi bởi KietLW9 on 28-03-2021 - 17:55 trong Bất đẳng thức và cực trị

 

2/cho a$\neq b\neq c$ Tìm min A=$\sum \frac{x^3-y^3}{(x-y)^3}$

 

Ta có đẳng thức: $\frac{x+y}{x-y}.\frac{y+z}{y-z}+\frac{y+z}{y-z}.\frac{z+x}{z-x}+\frac{z+x}{z-x}. \frac{x+y}{x-y} =-1$

Ta luôn có: $(\frac{x+y}{x-y}+\frac{y+z}{y-z}+\frac{z+x}{z-x})^2\geqslant 0$

$\Leftrightarrow \sum (\frac{x+y}{x-y})^2+2(\frac{x+y}{x-y}.\frac{y+z}{y-x}+\frac{y+z}{y-z}.\frac{z+x}{z-x}+\frac{z+x}{z-x}.\frac{x+y}{x-y})\geqslant 0$

Vậy $\sum (\frac{x+y}{x-y})^2\geqslant 2 $(*)

hay $\sum \frac{x^2+y^2}{(x-y)^2}\geqslant \frac{5}{2}$ (1)

Trừ 3 cho hai vế của (*), ta được:$ \sum ((\frac{x+y}{x-y})^2-1)\geqslant -1\Leftrightarrow \sum \frac{4xy}{(x-y)^2}\geqslant -1\Leftrightarrow  \sum \frac{xy}{(x-y)^2}\geqslant \frac{-1}{4}$ (2)

Cộng theo vế hai BĐT (1) và (2), ta được: $\sum \frac{x^2+y^2+xy}{(x-y)^2}\geqslant \frac{9}{4}\Leftrightarrow \sum \frac{(x-y)(x^2+y^2+xy)}{(x-y)^3}\geqslant \frac{9}{4}\Rightarrow Q.E.D$

Đẳng thức xảy ra khi $\frac{x+y}{x-y}+\frac{y+z}{y-z}+\frac{z+x}{z-x}=0$